Đến nội dung

Hình ảnh

i) $\sum\limits_{i=1}^{100}{a_{i}}^{2} > 10000$ ii) $ \sum\limits_{i=1}^{100}a_{i} < 300$


  • Please log in to reply
Chủ đề này có 5 trả lời

#1
ngtl

ngtl

    Trung sĩ

  • Thành viên
  • 131 Bài viết

Cho 100 số thực dương thỏa mãn điều kiện:
i) $\sum\limits_{i=1}^{100}{a_{i}}^{2} > 10000$
ii) $ \sum\limits_{i=1}^{100}a_{i} < 300$
Chứng minh rằng luôn tồn tại 3 số có tổng không nhỏ hơn 100.
 


Bài viết đã được chỉnh sửa nội dung bởi E. Galois: 24-05-2013 - 16:36

Càng học càng thấy mình ngu.
Không học lại thấy thông minh hơn người.

#2
PSW

PSW

    Những bài toán trong tuần

  • Quản trị
  • 493 Bài viết

Bài toán này thuộc Gameshow NHỮNG BÀI TOÁN TRONG TUẦN. Bài toán đã được công bố lại hơn 2 ngày nhưng chưa ai giải được. BTC đã đặt hoa hồng hi vọng  @};- cho bài toán này.

Hoa hồng hi vọng  @};- sẽ mang lại 50 điểm cho người đầu tiên giải đúng được bài toán này. Nếu hết ngày 28/05 mà vẫn không có ai giải được, BTC sẽ công bố bài toán khác, tuy nhiên hoa hồng hi vọng  @};- sẽ vẫn tồn tại cho đến khi có người giải được bài toán này.


1) Thể lệ
2) Danh sách các bài toán đã qua: 1-100, 101-200, 201-300, 301-400
Còn chờ gì nữa mà không tham gia! :luoi:

#3
ilovelife

ilovelife

    Sĩ quan

  • Thành viên
  • 371 Bài viết

Cho 100 số thực dương thỏa mãn điều kiện:
i) $\sum\limits_{i=1}^{100}{a_{i}}^{2} > 10000$
ii) $ \sum\limits_{i=1}^{100}a_{i} < 300$
Chứng minh rằng luôn tồn tại 3 số có tổng không nhỏ hơn 100.

Note: lời giải của em có hỗ trợ từ 1 người bạn và có hỗ trợ từ công nghệ.
 
Không mất tổng quát, giả sử $x_{i+1} \ge x_i$ và ta sẽ đi chứng minh $x_{100} + x_{99} + x_{98} \ge 100$.
Giả sử đầu bài sai, tức $\not\exists 3$ số có tổng $\ge 100 \implies S = x_{100} + x_{99} + x_{98} < 100\ \text{(iii)}$
Gọi $x_j \ge x_i \ge t \ge 0$
Xét $(x_i-t)^2+(x_j+t)^2 = x_i^2+x_j^2+2t(x_j-x_i+t) \ge x_i^2+x_j^2$ (vì $x_j - x_i + t \ge 0$)
$\implies$ nếu thay $x_i -t \to x_i,\ x_j +t \to x_j$ thì vẫn cho tổng $2$ số không đổi nhưng tổng bình phương tăng lên.
Ta cũng có thể thay $x_{100}, x_{99}, x_{98}$ bằng $a = \frac S3$ là trung bình cộng của $3$ số đó mà không ảnh hưởng tới các điều kiện bài toán. (với $a < \frac {100}{3}$)
 
Phần thuật toán:
 



$\boxed{1.}$ Gán $i = \min,\ j = \max$ (lưu ý: $0 < x_i, x_j < a$)

$\boxed{2.}$ Nếu $i \ge j \to \boxed {\text{stop}}$

$\boxed{3.}$ Còn không, $t = \min \{x_i, a-x_j\},\ x_i := x_i - t,\ x_j := x_j + t$. Như nói ở trên, khi thế $x_i -t \to x_i,\ x_j +t \to x_j$ thì vẫn cho tổng $2$ số không đổi nhưng tổng bình phương tăng lên do đó, ta được dãy mới và cũng cũng thoả điều kiện (i), (ii), (iii)

$\boxed{4.}$ Quay lại bước $\boxed 1$

 

Phần Output, và giải: 
Sau khi vòng lặp kết thúc, ta thu được: $0 = x_1 = x_2 =\ldots = x_{i-1}< x_i \le x_{i+1} = x_{i+2} = \ldots = x_{100} = a$
$\implies x_i + (100 - i)\cdot a < 300, x_i^2 + (100-i)\cdot a^2 > 10^4, 0 < x_i \le a < \frac {100} 3$
$\implies 10^4 - ab > (300 - b)\cdot a = ca^2 > 10^4 - b$ (với $b = x_i,\ c = 100 - i$)
$\implies b = 0 \lor a<1$
$\boxed{\text{TH1:}\ a<1}\implies 10^4 < b^2 + ca^2 < b+ca < 300 \implies$ vô lí
$\boxed{\text{TH2:}\ b=0} \implies c\cdot 10^4 < (ac)^2 < (300)^2 \implies c < 9 \implies 9a^2 > ca^2 > 10^4$
$\implies 3a > 100 \iff a > \frac {100} 3 \implies$ mâu thuẫn
Vậy điều giả sử là sai, đầu bài là đúng. Bài toán được chứng minh

 

Cách khác:

Giả sử $x_1 \ge x_2 \ge \ldots \ge x_{100}, S = x_1 + x_2 + x_3$

$$\implies x_1^2+x_2^2+x_3^2+ \ldots +x_{100}^2\\ \le x_1^2+x_2^2+x_3^2+\left\lfloor\frac{300-S}{x_3}\right\rfloor x_3^2+\left(300-\left\lfloor\frac{300-S}{x_3}\right\rfloor x_3\right)^2 \\ \le (S-2x_3)^2+2x_3^2 + \left\lfloor\frac{300-S}{x_3}\right\rfloor x_3^2+\left(300-\left\lfloor\frac{300-S}{x_3}\right\rfloor x_3\right)^2$$

 

Đi tắm đã, lúc nào em làm tiếp, bây giờ em "hơi" lười


Bài viết đã được chỉnh sửa nội dung bởi ilovelife: 14-06-2013 - 17:30

God made the integers, all else is the work of man.

People should not be afraid of their goverment, goverment should be afraid of their people.

 


#4
Valar Morghulis

Valar Morghulis

    Binh nhất

  • Thành viên
  • 24 Bài viết

Đây là một bài ứng dụng đơn giản của nguyên lý cực hạn.

Giả sử 100 số đó là $a_{1} \geq a_{2} \geq ... \geq a_{100} >0$.

Nếu như $a_{1} \geq 100$, thì $a_{1} + a_{2} + a_{3} > 100$.

Do đó, ta chỉ cần chứng minh với $a_{1} <100$.

Khi đó $100 - a_{1} >0, 100 - a_{2} >0, a_{1} -a_{3}\geq 0, a_{2} - a_{3} \geq 0$.

Vì vậy:

$100(a_{1} + a_{2} + a_{3}) \geq 100(a_{1} + a_{2} + a_{3}) - (100 - a_{1})(a_{1} -a_{3}) - (100 - a_{2})(a_{2} - a_{3}) = a_{1}^{2} + a_{2}^{2} + a_{3}( 300 - a_{1} - a_{2}) > a_{1}^{2} + a_{2}^{2} + a_{3}( a_{3} + a_{4} + ... + a_{100}) \geq a_{1}^{2} + a_{2}^{2} + a_{3}^{2} + ... + a_{100}^{2} > 10000.$

Suy ra, $a_{1} + a_{2} + a_{3} >100$

 

 

 

 

 

 

 

 

 

 

 



#5
holmes2013

holmes2013

    Trung sĩ

  • Thành viên
  • 189 Bài viết

Không mất tính tổng quát, giả sử $a_{1}\geq a_{2}\geq ...\geq a_{100}$

Trường hợp 1: $a_{1}\geq 100$

                      $\Rightarrow a_{1} + a_{2} + a_{3} \geq 100$

 

Trường hợp 2: $a_{1} \leq 100$

        Ta có:   $100\left ( a_{1} + a_{2} + a_{3}\right ) \geq 100\left ( a_{1} + a_{2} + a_{3}\right )-\left ( 100 - a_{1} \right )\left ( a_{1}-a_{3} \right ) - \left ( 100 - a_{2} \right )\left ( a_{2} -a_{3}\right )$

                    $= a_{1}^{2} + a_{2}^{2} + a_{3}\left ( 300 - a_{1} - a_{2} \right )$

                    $\geq a_{1}^{2} + a_{2}^{2} + a_{3}\left ( a_{3} + a_{4} + ... + a_{100}\right )$

                    $\geq a_{1}^{2} + a_{2}^{2} + ... + a_{100}^{2} \geq 10000$

            $\Rightarrow a_{1} + a_{2} + a_{3}\geq 100$



#6
PSW

PSW

    Những bài toán trong tuần

  • Quản trị
  • 493 Bài viết

Chấm bài:

Valar Morghulis: 50 điểm

 ilovelife: 5 điểm


1) Thể lệ
2) Danh sách các bài toán đã qua: 1-100, 101-200, 201-300, 301-400
Còn chờ gì nữa mà không tham gia! :luoi:




0 người đang xem chủ đề

0 thành viên, 0 khách, 0 thành viên ẩn danh